7
$\begingroup$

Consider the finite Boolean lattice $B_n$ of subsets of $[n]:=\lbrace 1,\dots,n\rbrace$ ordered by inclusion, let $1\leq j,k\leq n$ and consider the poset: $$A_{j,k}=\lbrace\emptyset\neq U\in B_n\mid (1,\dots,j)\nsubseteq U, (n-k,\dots,n)\nsubseteq U \rbrace$$ I want to compute the geometric realization for every $j,k$. Using the ideas of (Homotopy type of the geometric realization of a poset) I was able to prove the following:

If $j\leq n-k-1$ then: $$|A_{j,k}|=|((B_{j}\setminus\lbrace\hat{1}\rbrace)\times B_{n-j-k-1}\times (B_{k+1}\setminus\lbrace\hat{1}\rbrace))\setminus\lbrace\hat{0}\rbrace|=$$$$=|B_{j}\setminus\lbrace\hat{1},\hat{0}\rbrace|\ast| B_{n-j-k-1}\setminus\lbrace\hat{0}\rbrace|\ast |B_{k+1}\setminus\lbrace\hat{1},\hat{0}\rbrace|=$$$$=\mathbb{S}^{j-2}\ast | B_{n-j-k-1}\setminus\lbrace\hat{0}\rbrace|\ast \mathbb{S}^{k-1}=\mathbb{S}^{j+k-2}\ast | B_{n-j-k-1}\setminus\lbrace\hat{0}\rbrace|$$ Then, if $n=j+k+1~|A_{j,k}|=\mathbb{S}^{j+k-2}=\mathbb{S}^{n-3}$ and if $n\neq j+k+1~|A_{j,k}|$ is contractible.

But I don't know how we can prove it for $j\geq n-k$, since in this case the two sets $(1,\dots,j)$ and $(n-k,\dots,n)$ intersect, so I can't split $A_{j,k}$ into a direct product.

Any hint or help will be thanked.

Edit: Using the hints of @ChristopheLeuridan and @DavidWhite I did the following for the case $j\geq n-k$: First of all, we can decompose $A_{j,k}=A_{j,k}^1\cup A_{j,k}^2$ where: $$A_{j,k}^1=\lbrace\emptyset\neq U\in B_n\mid (n-k,\dots,j)\nsubseteq U\rbrace$$ $$A_{j,k}^2=\lbrace\emptyset\neq U\in B_n\mid (n-k,\dots,j)\subseteq U\text{ and }(1,\dots,n-k-1)\nsubseteq U,(j+1,\dots,n)\nsubseteq U\rbrace$$ We can compute the geometric realization of those two subposets: $$|A_{j,k}^1|=|((B_{j-n-k+1}\setminus\lbrace\hat{1}\rbrace)\times B_{2n-j+k-1})\setminus\lbrace\hat{0}\rbrace|=$$ $$=|B_{j-n-k+1}\setminus\lbrace\hat{0},\hat{1}\rbrace|\ast|B_{2n-j+k-1}\setminus\lbrace\hat{0}\rbrace|=\mathbb{S}^{j-n+k-3}\ast|B_{2n-j+k-1}\setminus\lbrace\hat{0}\rbrace|$$ Hence, if $2n-j+k-1=0~|A_{j,k}^1|=\mathbb{S}^{j-n+k-3}=\mathbb{S}^{n+2k-4}$ and if $2n-j+k-1\neq0~|A_{j,k}^1|$ is contractible, but it is easy to see that we can't have $2n-j+k-1=0$. On the other hand: $$|A_{j,k}^2|=|((B_{n-k-1}\setminus\lbrace\hat{1}\rbrace)\times(B_{n-j}\setminus\lbrace\hat{1}\rbrace))\setminus\lbrace \hat{0}\rbrace|=$$ $$=|B_{n-k-1}\setminus\lbrace\hat{0},\hat{1}\rbrace|\ast|B_{n-j}\setminus\lbrace\hat{0},\hat{1}\rbrace|=\mathbb{S}^{n-k-3}\ast\mathbb{S}^{n-j-2}=\mathbb{S}^{2n-k-j-4}$$ Hence, since those two posets are disjoint we have: $$|A_{j,k}|=|A_{j,k}^1|\cup|A_{j,k}^2|=\mathbb{S}^{2n-k-j-4}$$

$\endgroup$
7
  • $\begingroup$ If $j \ge n-k$, the condition can be written EITHER $\{n-k,...,j\} \not\subset U$ OR [$\{n-k,...,j\} \subset U$ and $\{1,...,n-k-1\} \not\subset U$ and $\{j+1,...,n\} \not\subset U$]. Does it help? $\endgroup$ Jun 24 at 20:26
  • $\begingroup$ @ChristopheLeuridan not really... Using your hint I can write my poset as the union of two posets. But in general I dont know any relations between the geometric realización of the union. $\endgroup$
    – Marcos
    Jun 25 at 7:36
  • $\begingroup$ Well, geometric realization is a left adjoint, and those preserve colimits. And the union is a colimit, since it's a coproduct. Does this help? math.stackexchange.com/questions/2108256/… $\endgroup$ Jun 25 at 11:48
  • $\begingroup$ @DavidWhite I guess that means that $|A\cup B|=|A|\cup|B|$ when they are disjoint, right? I'm not really confident with my cathegory theory knowledge, so I'm not totally sure. $\endgroup$
    – Marcos
    Jun 25 at 17:40
  • $\begingroup$ Yes, that's right. Perhaps that observation can help you. $\endgroup$ Jun 25 at 23:14

1 Answer 1

2
+50
$\begingroup$

Put $P_0=\{1,\dotsc,n-k-1\}$ and $P_1=\{n-k,\dotsc,j-1\}$ and $P_2=\{j,\dotsc,n\}$, so $[n]=P_0\amalg P_1\amalg P_2$ with each $P_i$ nonempty. Any subset $U$ can be decomposed as $\coprod_{i=0}^2U_i$ with $U_i\subseteq P_i$. You are looking at the space $$ A = \{(U_0,U_1,U_2): U_0\cup U_1\cup U_2\neq\emptyset, U_0\cup U_1\neq P_0\cup P_1, U_1\cup U_2\neq P_1\cup P_2\}. $$ This can be written as $B\cup C$, where \begin{align*} B &= \{(U_0,U_1,U_2): U_0\cup U_1\cup U_2\neq\emptyset,U_1\neq P_1\} \\ C &= \{(U_0,U_1,U_2): U_0\cup U_1\cup U_2\neq\emptyset,U_0\neq P_0,U_2\neq P_2\} \\ D &= B\cap C = \{(U_0,U_1,U_2): U_0\cup U_1\cup U_2\neq\emptyset,U_0\neq P_0,U_1\neq P_1,U_2\neq P_2\} \end{align*} We can define $f_0,f_1,f_2\colon B\to B$ by $f_0=\text{id}$ and $f_1(U_0,U_1,U_2)=(P_0,U_1,P_2)$ and $f_2(U_0,U_1,U_2)=(P_0,\emptyset,P_2)$. We have $f_0\leq f_1\geq f_2$, so the identity map $|f_0|\colon|B|\to|B|$ is homotopic to the constant map $|f_2|$, so $B$ is contractible. Similarly, we can define $g_0,g_1,g_2\colon C\to C$ by $g_0=\text{id}$ and $g_1(U_0,U_1,U_2)=(U_0,P_1,U_2)$ and $g_2(U_0,U_1,U_2)=(\emptyset,P_1,\emptyset)$, and use these to see that $|C|$ is contractible. We also have a pushout square of subcomplex inclusions $\require{AMScd}$ \begin{CD} |D| @>>> |B|\simeq * \\ @VVV @VVV\\ |C|\simeq * @>>> |A| \end{CD} which gives $|A|\simeq\Sigma|D|$. Now put $D_i=\{U_i : \emptyset\subset U_i\subset P_i\}$, so $|D_i|\simeq S^{|P_i|-2}$. It is easy to see that $|D|$ is the same as the join $|D_0|*|D_1|*|D_2|$, and $S^r*S^s\simeq S^{r+s+1}$, so $|D|\simeq S^{|P_0|+|P_1|+|P_2|-4}=S^{n-4}$ and $|A|\simeq\Sigma S^{n-4}=S^{n-3}$.

$\endgroup$
1
  • $\begingroup$ Thanks!! Can you tell me where I did a mistake in my attempt? Because I'm not able to see it. $\endgroup$
    – Marcos
    Jun 26 at 12:19

Your Answer

By clicking “Post Your Answer”, you agree to our terms of service and acknowledge that you have read and understand our privacy policy and code of conduct.

Not the answer you're looking for? Browse other questions tagged or ask your own question.